Difference between revisions of "2010 AMC 10B Problems/Problem 25"

(Redirected page to 2010 AMC 12B Problems/Problem 21)
(Tag: New redirect)
 
(40 intermediate revisions by 25 users not shown)
Line 1: Line 1:
There must be some polynomial <math>Q(x)</math> such that <math>P(x)-a=(x-1)(x-3)(x-5)(x-7)Q(x)</math>
+
#REDIRECT [[2010 AMC 12B Problems/Problem 21]]
 
 
Then, plugging in values of <math>2,4,6,8,</math> we get
 
 
 
<math>P(2)-a=(2-1)(2-3)(2-5)(2-7)Q(2) = -15Q(2) = -2a</math>
 
<math>P(4)-a=(4-1)(4-3)(4-5)(4-7)Q(4) = -15Q(4) = -2a</math>
 
<math>P(6)-a=(6-1)(6-3)(6-5)(6-7)Q(6) = -15Q(6) = -2a</math>
 
<math>P(8)-a=(8-1)(8-3)(8-5)(8-7)Q(8) = -15Q(8) = -2a</math>
 
 
 
<math>-2a=-15Q(2)=9Q(4)=-15Q(6)=105Q(8).</math>
 
Thus, the least value of <math>a</math> must be the <math>lcm(15,9,15,105)</math>.
 
Solving, we receive <math>315</math>, so our answer is <math> \boxed{\textbf{(B)}\ 315} </math>.
 

Latest revision as of 15:26, 17 June 2020